Difference between revisions of "2021 AMC 12B Problems/Problem 15"
(→Problem 15) |
(→Problem 20) |
||
Line 23: | Line 23: | ||
<math>\textbf{(A)} ~20 \qquad\textbf{(B)} ~21 \qquad\textbf{(C)} ~22 \qquad\textbf{(D)} ~23 \qquad\textbf{(E)} ~24</math> | <math>\textbf{(A)} ~20 \qquad\textbf{(B)} ~21 \qquad\textbf{(C)} ~22 \qquad\textbf{(D)} ~23 \qquad\textbf{(E)} ~24</math> | ||
+ | |||
+ | ==Solution== | ||
+ | |||
+ | Let <math>M</math> be the midpoint of <math>CD</math>. |
Revision as of 19:32, 11 February 2021
Problem 20
The figure is constructed from line segments, each of which has length . The area of pentagon can be written is , where and are positive integers. What is
Solution
Let be the midpoint of .